LSAT and Law School Admissions Forum

Get expert LSAT preparation and law school admissions advice from PowerScore Test Preparation.

 Administrator
PowerScore Staff
  • PowerScore Staff
  • Posts: 8916
  • Joined: Feb 02, 2011
|
#22799
Complete Question Explanation

Weaken-SN. The correct answer choice is (C)

In this stimulus, the author notes that high production costs are limiting which operas are made available to the public. Why is this the case? Because the high costs require the use of corporate sponsors, but those corporate sponsors only want to fund the most famous operas (if you think about this for a second, it makes some sense: would a corporation want to fund an opera no one has ever heard of or one that is famous and well-known? They probably get more mileage from the famous one, hence the preference there). But the author believes that the ticket purchasers should control the decision of what operas are produced, not these large corporations. So, the recommendation is made to reduce production budgets, which will then allow the public to see less famous operas.

There is a slew of conditional statements in this stimulus, but the key is to understand that the high costs necessitate a reliance on corporations, and that if the production budgets are lowered, the public will be able to see less famous operas.

The question stem asks you to Weaken the argument, which in this case will likely involve showing that even with lower budgets, the public still won't be able to see less famous operas (in other words, countering the final sentence of the stimulus).

Answer choice (A): The presence of the phrase "a few" provides a clear indication of the vagueness of this answer choice, which has no effect on the argumentation in the stimulus.

Answer choice (B): The author's argument does not consider the preferences of corporate sponsors—the current system shows that these sponsors obviously do prefer to sponsor famous operas. The argument from the stimulus regards the effects of removal of corporate sponsorships, regardless of corporate preference.

Answer choice (C): This is the correct answer choice, and it is a very tricky one. First, the answer intends to shows that once you remove the large corporate sponsors, then, even if the shows are supported exclusively by box-office receipts and donations from individuals, that the result is still that only the most famous operas are produced. So, in this sense, this answer negates the final sentence of the argument, which is why it is correct.

However, while the abstract relationship between this answer and the argument might be clear, that still does not answer the question of how this actually undermines the argument in direct terms, so let's discuss that point. I used to explain the mechanism that can underlie this answer in class as follows:
  • Let's say you are producing an opera, and you follow this advice, and you tell your corporate sponsors to take a hike. Instead, you decide to produce a relatively small and unknown opera that you always loved, and you do so with a much reduced budget. What this answer is saying is that when you make the tickets available to the general public, they see the opera name and don't recognize it, and choose not to buy tickets. So, even though we no longer have corporate sponsors, the demand is also no longer there because the new opera isn't well-known and doesn't draw people in. To get tickets sales, you've just discovered that you can't offer unknown operas and must instead offer something famous enough to draw people in
Seen from that angle, this answer accomplishes exactly what is discussed above: the use of just box office receipts (and thus the removal of corporate sponsorships) leads to a situation where the public does not get to see less famous operas. This counters the last sentence, making this answer correct.

Answer choice (D): Like incorrect answer choice (B) above, this answer is incorrect because it involves the actions of corporate sponsors, not the reaction of the opera system to reduction of reliance on corporate sponsorships.

Answer choice (E): The author's argument already considers this because the suggestion was to reduce budgets which would then allow for less dependence on corporations. The fact that other sources might raise less revenues does not weaken the assertion from the stimulus.
 GLMDYP
  • Posts: 100
  • Joined: Aug 19, 2013
|
#12426
Hi!
For this question, I cannot figure out why (C) is correct. I suppose famous plays should be more expensive, right? But (C) seems suggest that the less famous plays are more expensive. Why is this? This does not fit into the scenario at all.
Thanks!
 Emily Haney-Caron
PowerScore Staff
  • PowerScore Staff
  • Posts: 577
  • Joined: Jan 12, 2012
|
#12438
Hi GLMDYP,

Great question. Here we're trying to find the answer choice that will weaken the conclusion, which is: "If we reduce production budgets so that operas can be supported exclusively by box‑office receipts and donations from individuals, then the public will be able to see less famous operas." C does that by showing that EVEN IF operas are supported exclusively by box-office receipts and donations, opera companies will STILL only produce the most famous operas. In the world of this question, C weakens the argument.

This answer choice definitely feels counterintuitive to a lot of people - it seems like famous plays should be more expensive. But we aren't concerned with whether C seems logical given what we know about the world - we're only focused on the information provided to us in this question. Remember, the question says, "which of the following, if true, would weaken the argument? Therefore, we assume C is true, even if it doesn't seem very likely to us.

Does that help to explain it?
 JayI
  • Posts: 4
  • Joined: Jul 27, 2017
|
#37926
Hi,

I dont understand why answer choice E is not correct. The last sentence of the stimulus says that " ..so that operas can be supported by box office receipts and donations from individuals, then the public will be able to see less famous operas".

Since answer choice E says that wont be enough, it weakens the argument. Can you please let me know why this is incorrect? Thank you.
 AthenaDalton
PowerScore Staff
  • PowerScore Staff
  • Posts: 296
  • Joined: May 02, 2017
|
#37957
Hi Jay,

Great question!

The stimulus says that if we "reduce production budgets so that operas can be supported exclusively by box-office receipts and donations from individuals" then we can see less famous operas.

The argument assumes that the operas produced will be lower-budget affairs which won't require the same amount of financial support as the big-budget operas supported by corporate donations. So it's ok if individual donations don't match the level of corporation donations, as answer choice (E) indicates.

For example, producing a really famous opera like Carmen might cost $5 million and would require $1 million in box office receipts and $4 million in corporate donations. By contrast, producing Low Budget Indie Opera might only cost $2 million, requiring $1 million in box office sales and $1 million in donations.

Even if answer choice (E) is true -- that individual donations won't match corporate donations -- it will still be enough to stage an opera so long as production costs are cut.

I hope this clears things up. Good luck studying! :)
 Kaylahernandez
  • Posts: 3
  • Joined: Aug 29, 2017
|
#40771
I want to make sure my reasoning is clear in this question because at first it wasn't very clear to me how C weakened the stimulus (even though I understand why every other answer is wrong).
So I went back and reviewed the chapter and based on the lesson on weaken questions, answers that weaken an argument's conclusion will attack assumptions made by author. In this instance the author assumes that lowering the budgets will allow for less famous plays, so the correct weaken answer attacks that assumption by implying regardless of the lower budgets, without large corporate sponsorship opera companies would have to produce only the most famous plays. Is this correct? And would a correct prephrase be something like "lower budgets will still not allow for less famous plays"?
User avatar
 Dave Killoran
PowerScore Staff
  • PowerScore Staff
  • Posts: 5853
  • Joined: Mar 25, 2011
|
#40772
Hi Kayla,

Yes, you are thinking about this the right way! I've actually gone in and changed the explanation for the stimulus and answer (C) to hopefully make them even clearer, but this answer is definitely addressing the idea that lower budgets/non-corporate funded will not allow for less famous operas.

Good job!
 andriana.caban
  • Posts: 142
  • Joined: Jun 23, 2017
|
#63485
Emily Haney-Caron wrote:Hi GLMDYP,

Great question. Here we're trying to find the answer choice that will weaken the conclusion, which is: "If we reduce production budgets so that operas can be supported exclusively by box‑office receipts and donations from individuals, then the public will be able to see less famous operas." C does that by showing that EVEN IF operas are supported exclusively by box-office receipts and donations, opera companies will STILL only produce the most famous operas. In the world of this question, C weakens the argument.

This answer choice definitely feels counterintuitive to a lot of people - it seems like famous plays should be more expensive. But we aren't concerned with whether C seems logical given what we know about the world - we're only focused on the information provided to us in this question. Remember, the question says, "which of the following, if true, would weaken the argument? Therefore, we assume C is true, even if it doesn't seem very likely to us.

Does that help to explain it?
Can you explain/help me understand why the conclusion is ""If we reduce production budgets so that operas can be supported exclusively by box‑office receipts and donations from individuals, then the public will be able to see less famous operas". I thought it was the prior sentence, thanks!

Additionally, I prephrased two things that I thought would also weaken the sentence. Can you please tell me if I'm on the right track?

1. Large corporate sponsors are also ticket holders

2. Large corporate sponsors give the most donations, and productions cannot exists without their support.

Based on prephrase (2), I choose E. If individual sponsors can't match the funds obtained by large corporate sponsors, then the opera probably won't have enough money to continue running productions.

I was confused by answer choice C and its use of the phrase 'any but'. Can you please explain what this means and how it effects the sentence?

I notice that this is a reoccurring problem I encounter where I prephrase and then mistakenly choose the wrong answer choice because I'm trying to mold the choice to my prephrase. Do you have any tips so I can avoid doing this in the future?
 Brook Miscoski
PowerScore Staff
  • PowerScore Staff
  • Posts: 418
  • Joined: Sep 13, 2018
|
#64274
Adriana,

An intermediate conclusion is: "Determining which operas will be produced should rest only with ticket purchasers at the box office, not with large corporate sponsors." This leads to the last sentence.

Your prephrases are good, although it is always worth keeping in mind that the LSAT test writers can weaken the argument in a way that we didn't expect. So your conceptual recognition that the weakness has something to do with the impact of larger sponsors is what is important.

The phrase "any but" goes with other words. "Could not afford...any but" is abysmally poor word choice for expressing "Can only afford." The LSAT test writers want to be sure that you can be an attorney, which involves reading dense and poorly written reprehensible drivel by people convinced they are gifted thinkers and writers.
 yusrak
  • Posts: 22
  • Joined: Mar 19, 2020
|
#75057
Hi Powerscore,

I struggled with identifying the conclusion in this stimulus. Whenever the stimulus does not have a conclusion indicator, yet somehow I intuitively know that it is an argument and not a fact set, I try to quickly organize and assess which ideas provide foundational support for purpose of the argument. But that didn't work, so I just prephrased that the audience are not interested in less famous operas and quickly moved on to the questions.

After reviewing this question, I wrote out conditional statements to help me figure out the conclusion:

High cost → limit options → rely on corporations → most famous
Choice → audience & not corporations
Reduce costs and fund through tickets/donations → audience will have a choice

I organized it in this way because I know now (after reading this forum) that the first 2 sets of conditional relationships provide support for the conclusion, but I don't understand why. I am really struggling with identifying conclusions that exist without indicators, and even if I do identify the conclusion without the indicator words, I can't grasp why it is the conclusion. I would appreciate any articles, blogs or problem sets to help with this!

Best,
Yusra

Get the most out of your LSAT Prep Plus subscription.

Analyze and track your performance with our Testing and Analytics Package.